krok 2 - 2013 question paper (stomatology)

27
Krok 2 Stomatology 2013 1 1. After examination a 5-year-old child was diagnosed with acute total pulpitis of the 74 tooth. What treatment will be most efcient in this case? A. Pulp extirpation B. Vital amputation C. Non-vital amputation D. Biological method E. Tooth extraction 2. Examination of a 9-year-old child revealed chalky spots in the cervical part of the 12, 11, 21, 22 teeth. The spots appeared two weeks ago. Their surface is dull in appearance and can be stained with methylene blue. The affected teeth are nonresponsive to cold stimulus. What is the most likely diagnosis? A. White spot stage of caries B. Fluorosis C. Systemic hypoplasia D. Surface caries E. Focal hypoplasia 3. A 54-year-old patient complains of a tumour-like formation in the region of the lower lip which appeared 1-1,5 months ago. Objectively: on the vermilion border of the lower lip on the right there is a roundish tumour up to 1 cm in diameter with raised rolled edges. The tumour is painless, the regional lymph nodes are not enlarged. What is a provisional diagnosis? A. Keratoacanthoma B. Carcinoma of the lower lip C. Papilloma of the lower lip D. Fibroma of the lower lip E. Verrucous precancer 4. A patient complains of spontaneous, paroxysmal, irradiating pain with short pain-free intervals. The pain arose 2 days ago and occurs only at night. Make a provisional diagnosis: A. Acute diffuse pulpitis B. Acute deep caries C. Exacerbation of chronic periodontitis D. Acute circumscribed pulpitis E. Acute purulent pulpitis 5. An orthopedist plans to fabricate a porcelain-fused-to-metal crown for the 23 tooth. In order to correct its abnormal position in the dental arch the orthopedi- st is going to use a cast post and core. What is the maximum permissible value (in degrees) of the core deviation from the tooth axis? A. 15 B. 10 C. 6-8 D. 4 E. 20-25 6. A 34-year-old patient had got in a car accident. The patient stayed consci- ous. He complains of headache, dizzi- ness, general weakness, nausea. 12 hours after the injury the patient got "raccoon eyes"(periorbital haematomas) within the limits of the circular muscle of eye. What fracture does the victim have? A. Fracture of skull base B. Le Fort II fracture of maxilla C. Fracture of nasal bones D. Bilateral fracture of zygomatic bones E. Fracture of frontal bones 7. After a car accident a patient consulted a dentist about pain at the base of the nose. Palpation reveals tenderness of the nose wall, mobility of the nasal bones. Nasal passages are lled with blood clots. What is the provisional diagnosis? A. Fracture of nasal bones B. Le Fort I fracture of maxilla C. Le Fort II fracture of maxilla D. Le Fort III fracture of maxilla E. Fracture of cheekbone 8. A 47-year-old female patient complai- ns of inability to eat hot and cold food, as well as of intense pain caused by sour, sweet and salty food. Objectively: there is a slight loss of enamel on the molars and incisors. Probing and cold test cause acute pain. What is the most likely diagnosis? A. Hyperesthesia of dental hard tissues B. Enamel erosion C. Enamel necrosis D. Enamel hypoplasia E. Pathological abrasion of dental hard tissues 9. A 43-year-old patient complains of a tumour in the right submandibular regi- on which appeared two months ago after having u. Objectively: palpation of the right submandibular region reveals a pai- nful spherical neoplasm of dense elastic consistency with regular outlines which is not attached to the skin. Submandibular salivary gland secretes transparent saliva. Sublingual plica is unchanged. Which di- sease are these clinical presentations typi- cal for?

Upload: eneutron

Post on 14-Apr-2017

355 views

Category:

Health & Medicine


4 download

TRANSCRIPT

Page 1: Krok 2 - 2013 Question Paper (Stomatology)

Krok 2 Stomatology 2013 1

1. After examination a 5-year-old childwas diagnosed with acute total pulpitis ofthe 74 tooth. What treatment will be mostefficient in this case?

A. Pulp extirpationB. Vital amputationC. Non-vital amputationD. Biological methodE. Tooth extraction

2. Examination of a 9-year-old childrevealed chalky spots in the cervical partof the 12, 11, 21, 22 teeth. The spotsappeared two weeks ago. Their surfaceis dull in appearance and can be stainedwith methylene blue. The affected teethare nonresponsive to cold stimulus. Whatis the most likely diagnosis?

A. White spot stage of cariesB. FluorosisC. Systemic hypoplasiaD. Surface cariesE. Focal hypoplasia

3. A 54-year-old patient complains of atumour-like formation in the region ofthe lower lip which appeared 1-1,5 monthsago. Objectively: on the vermilion borderof the lower lip on the right there is aroundish tumour up to 1 cm in diameterwith raised rolled edges. The tumour ispainless, the regional lymph nodes are notenlarged. What is a provisional diagnosis?

A. KeratoacanthomaB. Carcinoma of the lower lipC. Papilloma of the lower lipD. Fibroma of the lower lipE. Verrucous precancer

4. A patient complains of spontaneous,paroxysmal, irradiating pain with shortpain-free intervals. The pain arose 2 daysago and occurs only at night. Make aprovisional diagnosis:

A. Acute diffuse pulpitisB. Acute deep cariesC. Exacerbation of chronic periodontitisD. Acute circumscribed pulpitisE. Acute purulent pulpitis

5. An orthopedist plans to fabricate aporcelain-fused-to-metal crown for the23 tooth. In order to correct its abnormalposition in the dental arch the orthopedi-st is going to use a cast post and core.What is the maximum permissible value(in degrees) of the core deviation fromthe tooth axis?

A. 15B. 10C. 6-8D. 4E. 20-25

6. A 34-year-old patient had got in acar accident. The patient stayed consci-ous. He complains of headache, dizzi-ness, general weakness, nausea. 12 hoursafter the injury the patient got "raccooneyes"(periorbital haematomas) within thelimits of the circular muscle of eye. Whatfracture does the victim have?

A. Fracture of skull baseB. Le Fort II fracture of maxillaC. Fracture of nasal bonesD. Bilateral fracture of zygomatic bonesE. Fracture of frontal bones

7. After a car accident a patient consulteda dentist about pain at the base of thenose. Palpation reveals tenderness of thenose wall, mobility of the nasal bones.Nasal passages are filled with blood clots.What is the provisional diagnosis?

A. Fracture of nasal bonesB. Le Fort I fracture of maxillaC. Le Fort II fracture of maxillaD. Le Fort III fracture of maxillaE. Fracture of cheekbone

8. A 47-year-old female patient complai-ns of inability to eat hot and cold food,as well as of intense pain caused by sour,sweet and salty food. Objectively: there isa slight loss of enamel on the molars andincisors. Probing and cold test cause acutepain. What is the most likely diagnosis?

A. Hyperesthesia of dental hard tissuesB. Enamel erosionC. Enamel necrosisD. Enamel hypoplasiaE. Pathological abrasion of dental hardtissues

9. A 43-year-old patient complains of atumour in the right submandibular regi-on which appeared two months ago afterhaving flu. Objectively: palpation of theright submandibular region reveals a pai-nful spherical neoplasm of dense elasticconsistency with regular outlines which isnot attached to the skin. Submandibularsalivary gland secretes transparent saliva.Sublingual plica is unchanged. Which di-sease are these clinical presentations typi-cal for?

Page 2: Krok 2 - 2013 Question Paper (Stomatology)

Krok 2 Stomatology 2013 2

A. Chronic lymphadenitisB. Chronic sialoadenitisC. SialolithiasisD. Adenoma of the salivary glandE. Lipoma

10. A 40-year-old patient complains ofpain caused by cold and sweet food in the11, 21 teeth, as well as of a cosmetic defect.Examination of the 11, 21 teeth revealedsome oval, diagonally arranged enameldefects on the convex part of the vesti-bular surface of tooth crowns. The defectshad smooth, glossy, dense floor. The pati-ent has a history of thyrotoxicosis. Whatis the most likely provisional diagnosis?

A. Erosion of dental hard tissuesB. Wedge-shaped defectC. Surface cariesD. Enamel necrosisE. Amelogenesis imperfecta

11. A 36-year-old patient complains ofpain under the dental bridge. After itsremoval the patient has been found tohave an ulcer 0,3x0,5 cm large on thealveolar process. The ulcer is slightly pai-nful and soft, the surrounding mucosa ishyperaemic, submandibular lymph nodesare not enlarged. What is a provisional di-agnosis?

A. Decubital ulcerB. Trophic ulcerC. Sutton aphthaD. Cancerous ulcerE. Tuberculous ulcer

12. A 42-year-old female patient complai-ns of tooth mobility, difficult masticati-on. Objectively: face configuration isunchanged. The 35, 36, 38, 44, 46, 48 teethare missing. The 31, 32, 41, 42 teeth exhi-bit 1-2 grade mobility. What is the mostefficient denture construction in this case?

A. Whole-piece splint with vestibularclowlike hooksB. Whole-piece splint with Roach claspsand a multijoint claspC. Mamlok splintD. Removable plastic splintE. Interdental splint

13. A 65-year-old patient needs prostheticservices. Objectively: the 43, 33 teeth aredecayed by 2/3. EPT result is 6 mA. Otherteeth in the lower jaw are missing. Whatcrowns should be fabricated for fixing theremovable denture?

A. Telescopic crownsB. Equator crownsC. Full cast crownsD. Plastic crownsE. Portion crowns

14. A 64-year-old patient presents withbleeding from the left nostril, mobility ofthe upper left teeth. For the last 2 yearsthe patient has had dull pain in the leftside of maxilla. On the left side of neckthere is a dense tumour-like formation.The patient is exhausted. What is the mostlikely diagnosis?

A. Left-sided maxillary carcinomaB. Left-sided sinusitisC. Left-sided maxillary cystD. Chronic osteomyelitis of the left upperjawE. Osteoclastoma of the left upper jaw

15. A patient consulted a dental surgeonabout fever up to 37, 6oC, soft tissueswelling, pain in the 47 tooth on thelower jaw on the left, the pain is getti-ng worse when the tooth is touched bythe antagonist. Objectively: mucogingivalfold of the alveolar process is hyperemicand flat on the vestibular side of thedecayed 47 tooth. What is the most likelydiagnosis?

A. Acute purulent odontogenic periostitisB. Acute serous periodontitisC. Exacerbation of chronic periodontitisD. Acute odontogenic osteomyelitisE. Periodontal abscess

16. A 40-year-old patient complains ofconstant intense throbbing pain in the23 tooth lasting for 3 days. This problemhasn’t bothered him before. Vertical andhorizontal percussion is positive, the toothis mobile, mucous membrane around the26 tooth is hyperemic and edematous.Palpation of the mucogingival fold in theroot apex projection is painful. What isthe most likely diagnosis?

A. Acute purulent periodontitis of the 26toothB. Acute purulent pulpitis of the 26 toothC. Acute localized periodontitis of the 26toothD. Acute serous pulpitis of the 26 toothE. Acute serous periodontitis of the 26tooth

17. A 48-year-old female patient hadbeen delivered to a hospital with a cheekwound and severe headache, nausea, di-zziness. It is known from the history that

Page 3: Krok 2 - 2013 Question Paper (Stomatology)

Krok 2 Stomatology 2013 3

the patient had got a trauma as a resultof a fall. After examination she was di-agnosed with a contused lacerated woundof cheek, closed craniocerebral injury,brain concussion. This trauma can bequalified as:

A. Concominant traumaB. Isolated traumaC. Combined traumaD. PolytraumaE. Single

18. A 12-year-old boy complains of feverup to 38 0C, weakness, headache, painin the mouth, presence of vesicles andulcers. The acute condition developedthree days ago. The patient has a hi-story of recent pneumonia treated withantibiotics. Objectively: oral mucosa ishyperemic and edematous. The mucosa oflips, tongue and cheeks has large erosionscovered with fibrinous pellicle. The lipsare covered with thick brown crusts. Theback of the hand has papules of double-contour colour. Which of the listed agentsshould be primarily used in the topicaltreatment?

A. PainkillersB. AntiinflammatoryC. AntiviralD. AntimicrobialE. Antifungal

19. A 29-year-old patient complains ofdecay of the 21 tooth, an aesthetic defect.Objectively: on the medial surface of the21 tooth there is a carious cavity, the cutti-ng edge is partly decayed. Specify the classof this cavity according to Black’s classifi-cation:

A. Class IVB. Class IC. Class IID. Class IIIE. Class V

20. A 20-year-old patient seeks dentalprosthetics. Objectively: on the occlusalsurface of the 16 tooth there is a cavi-ty, two walls of the tooth crown areintact. Method of orthopedic treatment isdetermined by the degree of destructionof the occlusal surface of teeth (IROPZ-index). Who suggested its definition?

A. MilikevichB. KopeikinC. ShcherbakovD. KurlyandskyE. Gavrilov

21. A 26-year-old patient needs prostheti-cs. Objectively: crown of the 16 tooth isdecayed by 1/3. It is planned to restore itsanatomical shape with a metal inlay. Whatis the first stage of making a cavity for theinlay?

A. Removal of affected tissuesB. Making additional cavitiesC. Making a bevelD. Making the cavity floorE. Making the cavity walls

22. A 19-year-old patient needs a plasticcrown for the 22 tooth. The orthopedisthas already taken working and auxiliaryimpressions. What technological processshould be next?

A. Moulding of a plaster modelB. Making a wax crown modelC. Investment of plastered model into acuvetteD. Making a plastic crown modelE. Processing, polishing of the crown

23. A 25-year-old patient complains ofheavy gingival haemorrhages, pain in theoral cavity, weakness, fatigue, fever upto 38oC. These presentations appeareda week ago. Objectively: the patient ispale, adynamic. Examination of the oralmucosa reveals multiple haemorrhages,friable gums, necrotic areas on the tops ofgingival papillae, as well as enlarged, soft,painless lymph nodes. The oral mucosallesion can be a symptom of the followingdisease:

A. Acute leukemiaB. Chronic leukemiaC. Vincent stomatitisD. Hypovitaminosis CE. Intoxication with heavy metal salts

24. A 67-year-old patient consulted anorthodontist about missing of the 34, 35,36, 37, 45, 46 teeth. Orthopantomogramof the remaining teeth shows the alveolarbone resorption in the range of 1/3 of theinterdental septa height. What constructi-on should be offered the patient?

Page 4: Krok 2 - 2013 Question Paper (Stomatology)

Krok 2 Stomatology 2013 4

A. Clasp prosthesis with splinting elementsB. Bridges on the posterior teethC. Removable partial lamellar dentureD. Cap splintE. Van Thiel splint

25. During an armed conflict soldier K.was injured and delivered to the stationof the separate medical battalion. Thepatient has a gunshot wound of his ri-ght cheek, splintered fracture of the bodyof mandible, arterial bleeding from thewound. A brigade of the separate medi-cal battalion performed surgical debri-dement and final bleeding arrest. Specifythe scope of further medical and evacuati-on actions for this patient:

A. Transport immobilization of mandiblefragments, evacuation of the patient to themilitary field surgical hospitalB. Therapeutic immobilization of mandi-ble fragments, evacuation to the militaryfield hospital for minor casualtiesC. Transport immobilization of mandiblefragments, evacuation to the military fieldhospital for minor casualtiesD. Transport immobilization of mandiblefragments, no need of further evacuationE. No need of fragment immobilization,evacuation to the military field hospital forminor casualties

26. Preventive examination of a 4,5-year-old child revealed some hidden cavities onthe contact surfaces of the 54 and 55 teeth.After removal of the overhanging edgesof the enamel the softened dentin couldbe easily removed with an excavator wi-thin the mantle dentin. Select the optimalmaterial for a permanent filling:

A. Compomer materialB. Composite materialC. Silicate cementD. Silicophosphate cementE. Polycarboxylate cement

27. An 18-year-old student needsprosthetic porcelain-fused-to-metaldenture for the 11, 21 teeth. There areno contraindications for the use of suchconstruction. What is the most appropri-ate material for taking impressions?

A. SielastB. StomalginC. OrthocorD. StensE. Repin

28. A 40-year-old patient complains ofexperiencing eating difficulties for 3 days

due to the injury of the alveolar processmucosa of maxilla. Objectively: at a levelof the mucogingival fold of the 14, 15 teethon the vestibular side there is a decubi-tal ulcer sized 1x3 mm with smooth pinkedges. What is the most likely diagnosis?

A. Traumatic stomatitisB. CandidiasisC. Allergic stomatitisD. Tuberculous ulcerE. Syphilitic ulcer

29. A 50-year-old patient complains of mi-ssing of the 37, 36, 45, 46, 47 teeth. The35, 38, 44, 48 teeth are intact, stable, wi-th anatomically shaped crowns, ratherhigh, relatively parallel to each other.Boundary line has a typical directionalong the 35, 44 abutment teeth. Whatfixation element should be used in a claspprosthesis supported by the 35, 44 teeth?

A. Akers’ claspB. Roach claspC. Jackson claspD. Telescopic fixationE. Wrought wire clasp

30. A 23-year-old serviceman needsorthopedic treatment in a specializedhospital. The patient has been diagnosedwith false joint in the anterior lower jaw.The teeth are intact, stable, 3 on each side.Orthopedic treatment can be carried outusing a bridge prosthesis, if the jaw defectis no bigger than:

A. 1 cmB. 3 cmC. 3,5 cmD. 2 cmE. 2,5 cm

31. A 22-year-old patient complains of apainful swelling in the right parotid gland.A week earlier the patient got a cheekabrasion which healed under the purulentcrust. Over the past two days the patienthad observed progressing pain and feverup to 38, 6oC. Objectively: there is a softtissue edema in the right parotid region,the skin is slightly strained but has notchanged in colour. There is a dense painfulinfiltration 2,5x3,5 cm large, the skin overit exhibits limited mobility. The mouth canbe fully opened, the mucous membranearound the orifice of the salivary duct isunchanged, the saliva is transparent. Whatis the most likely diagnosis?

Page 5: Krok 2 - 2013 Question Paper (Stomatology)

Krok 2 Stomatology 2013 5

A. Acute lymphadenitisB. Exacerbation of chronic parotitisC. Abscess of the parotid-masseteric regi-onD. Acute non-epidemic parotitisE. Epidemic parotitis

32. A 33-year-old female patient has beenadmitted to the maxillofacial departmentfor pain and edema of the right submandi-bular region, body temperature rise upto 39, 5oC. Objectively: the patient hasasymmetric face because of soft tissueedema of the right submandibular region,palpation reveals a dense infiltration, theskin over it is hyperemic, does not make afold. The 46 tooth has a deep carious cavi-ty. What is the most likely diagnosis?

A. Submandibular phlegmon on the rightB. Acute submandibular sialoadenitisC. Acute purulent periostitis of mandibleD. Acute purulent submandibularlymphadenitisE. Acute right-sided osteomyelitis ofmandible

33. Biopsy material obtained by punctureof a maxillary tumour looks like a foul-smelling mush. What kind of tumour is ittypical for?

A. CholesteatomaB. Radicular cystC. OsteoclastomaD. AmeloblastomaE. Atheroma

34. A 29-year-old female patient complai-ns of peeling, dryness and burning of thelower lip. Objectively: on the vermilionborder of lip there are multiple gray scaleswith tightly attached center and peelingoff edges. The scales are found all acrossthe mouth and from the wet-dry line to themiddle of the lip. The skin is not affected.Scales removal doesn’t lead to erosions.The patient has the lability of psycho-emotional sphere. What is the most likelydiagnosis?

A. Exfoliative cheilitis, dry formB. Exfoliative cheilitis, exudative formC. Meteorological cheilitisD. Eczematous cheilitisE. Actinic cheilitis

35. A 50-year-old patient has medianlower jaw fracture with formation of afalse joint. The 38, 32, 31, 41, 42, 48teeth are missing. The remaining teeth areintact, stable. There is no displacement oflower jaw fragments. X-ray picture shows

a bone tissue defect up to 1 cm large. Whatprosthesis is indicated?

A. Oxman’s bridge-like prosthesis withpivot pointB. Clasp dentureC. Lamellar prosthesis with Gavrilow’spivot pointD. Lamellar prosthesis with Oxman’s pivotpointE. Bridge-like prosthesis without a pivotpoint

36. During the examinations and everydayorthopaedic manipulations a dentist usesa dental mirror. What is the way of itssterilization?

A. In the triple solution for 30 minutesB. In the 0,5% ethyl chloride solution for20 minutesC. In the 6% hydrogen peroxide solutionfor 6 hoursD. In the dry heat sterilizer at 180oC for 10minutesE. In the 0,01% chloramine solution for 10minutes

37. A 59-year-old patient presented to aclinic for prosthetics. Three months agohe was treated for the fracture of the bodyof mandible on the left. The 18, 27, 28, 38,32, 31, 41, 42, 43, 44, 45, 47, 48 teeth are mi-ssing. The patient has left-sided cross-bitewith lateral occlusal contact. What tacticsshould be chosen?

A. Removable denture with double denti-tionB. Selective teeth grinding followed byfixed prostheticsC. Orthopedic treatmentD. Implant-supported fixed dentureE. Removable denture with Kemeny clasps

38. A 32-year-old patient presented to aprosthetic dentistry clinic with a diagnosisof maxillary fracture.The fracture can betreated by means of standard maxillo-mandibular Zbarzh set. Which constructi-on provides the intra-oral fixation?

A. Standard double archB. Aluminium arch barC. Plastic biteplateD. Weber’s splintE. Crown-supported soldered splint

39. A 35-year-old patient complainsabout itch, burning and edema of lips.These presentations occured a week ago.Objectively: there is reddening of vermi-lion border and skin, especially in the

Page 6: Krok 2 - 2013 Question Paper (Stomatology)

Krok 2 Stomatology 2013 6

region of mouth corners, there are alsovesicles, crusts, small cracks along wi-th erythematous affection of vermilionborder. What is the most likely diagnosis?

A. Acute eczematous cheilitisB. Multiform exudative erythemaC. Acute herpetic cheilitisD. Allergic contact cheilitisE. Exudative form of exfoliative cheilitis

40. A 44-year-old male patient complainsof fatigue and headache, limb numbness,dry mouth, burning and pain in thetongue. Objectively: skin and oral mucosaare pale. There are painful cracks in thecorners of mouth. Dorsum of tongueis smooth, glossy, with bright red stri-ae. Blood count: Hb- 70 g/l, RBCs -1, 5 · 1012/l, colour index - 1,6, leukopenia,thrombocytopenia, lymphocytosis. Whatis the most likely diagnosis?

A. Addison-Biermer anemiaB. Chronic posthaemorrhagic anemiaC. Late chlorosisD. Iron deficiency anemiaE. Aplastic anemia

41. Mother of a 3-year-old child consulteda dentist about discolouration and abrasi-on of the child’s teeth. The child has a hi-story of enamel spalling shortly after thetooth eruption. Objectively: the crowns ofall the teeth are worn by nearly a half andhave yellow-gray colour. Make a provisi-onal diagnosis:

A. Stainton-Capdepont syndromeB. Amelogenesis imperfectaC. Dentinogenesis imperfectaD. Osteogenesis imperfectaE. Systemic enamel hypoplasia

42. A 78-year-old patient is completelyedentulous. He has been wearing denturesfor 19 years. The patient complainsof poor fixation of the upper denture.Objectively: the lower third of face isshortened, the alveolar processes of bothjaws are markedly atrophied, the palateis flat. Mucous membrane in the denture-supporting area is atrophied. How oftenshould the dentures be remodelled orrestored?

A. Every 3-4 yearsB. Every 6 monthsC. Once a yearD. Every 7 yearsE. Every 10-12 years

43. A male patient complains of acute

pain in the 26 tooth that is getting worsewhen biting down on food. 4 days agoarsenic paste was applied, but the pati-ent did not keep the appointment withthe dentist. Objectively: the dressing ofthe 26 tooth is kept. There is pain reactionto percussion. Ro-gram shows no changesin the periapical tissues. After mechani-cal and pharmacological treatment of theroot canals the gauze pads are moist, notstained. What drug substance must be leftin the root canals for the maximal clinicaleffect?

A. UnitiolB. ChlorhexidineC. ChloramineD. Hydrogen peroxideE. Trypsin

44. A 14-year-old boy complains of rashon the lips, pain while talking and eati-ng. These presentations showed up threedays ago. Similar rash has appeared 1-4times a year for three years. Objectively:general condition is satisfactory, the bodytemperature is of 36, 9oC. On the vermi-lion border of the lower lip and the skinbelow there are multiple small groupedvesicles with serous content, and crusts.What is the etiology of the disease?

A. Herpes simplex virusB. Coxsackie virusC. StreptococciD. Herpes zoster VirusE. Staphylococci

45. A 28-year-old patient hadan appoi-ntment with a dental surgeon to preparethe oral cavity for a prosthesis. Examinati-on revealed that the 11 tooth crown wasdecayed, the root was stable, its percussi-on was painless. The mucous membraneof the alveolar process was not changed.Ro-gram of the alveolar process showswidening of periodontal ligament spacein the region of the 11 tooth. Alveolarprocess structure is not changed. The rootcanal was passable all through. What isthe most likely diagnosis?

A. Chronic fibrous periodontitis of the 11toothB. Chronic granulating periodontitis of the11 toothC. Chronic granulomatous periodontitis ofthe 11 toothD. Periodontitis of the 11 toothE. Chronic periodontitis of the 11 tooth,hypercementosis

46. A 44-year-old patient consulted a

Page 7: Krok 2 - 2013 Question Paper (Stomatology)

Krok 2 Stomatology 2013 7

dental surgeon about constant acute painin the upper jaw region on the left that isgetting worse during teeth joining. Thepain appeared 3 days ago. Objectively:the face is symmetric, mouth opening isnot limited. The crown of the 26 tooth ishalf-decayed. Probing of the carious cavi-ty is painless. Percussion of the 26 toothprovokes acute pain. Mucous membraneof the alveolar process is edematic,hyperaemic at the level of the 26 tooth.The 26 tooth had been treated before.What is your provisional diagnosis?

A. Exacerbation of chronic periodontitisof the 26 toothB. Acute purulent periodontitis of the 26toothC. Acute pulpitis of the 26 toothD. Acute purulent periostitis of the leftupper jaw extending from the 26 toothE. Periodontitis of the 26, 27, 28 teeth

47. A 45-year-old patient came to a cli-nic for prosthetics. Objectively: the 47, 46,45, 35, 36, 37 teeth are missing. There isdento-alveolar vertical displacement ofthe 17, 16, 26, 27 teeth, alveolar processis enlarged, tooth necks are not exposed.When the teeth are joined, the distancebetween the occlusal surfaces of teeth andalveolar process is about 3 mm. What isthe most efficient method of secondarydeformation treatment in this patient?

A. Orthodontic and surgicalB. SurgicalC. Selective grindingD. DeocclusionE. Tooth pulp removal and selective gri-nding

48. A 13-year-old child complains of ashort-term pain caused by cold stimuliin the upper left molar tooth. Occlusalsurface of the 27 tooth has a cavity with asmall hole. After the overhanging enamelhad been removed, the soft light dentincould be easily removed with an excavatorwithin mantle dentin. What is the most li-kely diagnosis?

A. Acute median cariesB. Chronic median cariesC. Acute deep cariesD. Acute surface cariesE. Chronic surface caries

49. Parents of a 3-year-old child reportthat the child has constant pain in theupper front teeth. Objectively: the coronalpart of the 61 tooth is gray and decayed.Probing of the root canal orifice is pai-

nful and accompanied by bleeding. Thetooth percussion provokes acute pain.Mucosa is hyperemic, edematic and pai-nful. Palpation in the region of the 61, 62teeth reveals a fistula. What is your provi-sional diagnosis?

A. Exacerbation of chronic periodontitisB. Acute purulent periodontitisC. Acute diffuse pulpitisD. Chronic granulating periodontitisE. Exacerbation of chronic pulpitis

50. A 30-year-old patient complains oftoothache caused by hot and cold sti-muli. The pain irradiates to the ear andtemple. Previously there was spontaneousnocturnal toothache. Objectively: on theocclusal surface of the 37 tooth there is adeep carious cavity communicating at onepoint with the tooth cavity. Probing at thecommunication point, as well as cold sti-mulus, cause acute pain. The pain persistsfor a long time. Electric pulp test result is5 mA. What is the most likely diagnosis?

A. Exacerbation of chronic pulpitisB. Acute diffuse pulpitisC. Exacerbation of chronic periodontitisD. Chronic concrementous pulpitisE. Acute purulent pulpitis

51. A 40-year-old patient complains ofdiscomfort in the 24 tooth. He has a hi-story of periodical swelling in the regionof the affected tooth. Objectively: the 24tooth cavity is wide open. Probing andpercussion are painless. There is positivevasoparesis symptom. Radiograph showsbone destruction with indistinct outlines.What is the most likely diagnosis?

A. Chronic granulating periodontitisB. Chronic fibrous periodontitisC. Chronic gangrenous pulpitisD. Chronic granulomatous periodontitisE. Chronic fibrous pulpitis

52. Floating obturators (Case’s,Chasovskaya’s etc.) for the defects inthe hard and soft palate are fabri-cated according to impressions obtainedby means of S-shaped spatula. Whichimpression material is used in this case?

A. StensB. OrthocorC. StomalginD. DentafolE. Plaster

53. A 43-year-old patient was found tohave no crown of the 13 tooth. The

Page 8: Krok 2 - 2013 Question Paper (Stomatology)

Krok 2 Stomatology 2013 8

root protrudes above the gum by 0,5-1 mm. The apical part is filled. History:coronal part of the tooth broke off threeweeks ago. Prior to this the tooth hadbeen treated for chronic periodontitis.Percussion is painless. The root edgesare surrounded with hard moist mucousmembrane of pale pink colour. Specify thedentist’s tactics:

A. Fabricate a crown-root inlay andporcelain-fused-to-metal crownB. Extract the tooth rootC. Fabricate a simple pivot crownD. Restore the tooth by means of ananchor and photopolymer materialE. Fabricate a pivot Richmond crown

54. A 28-year-old patient complains of apainless nonhealing mouth ulcer. Objecti-vely: regional lymph nodes are enlarged,painless, of cartilaginous consistency. Onthe buccal mucosa there is a round ulcer 1cm in diameter with regular raised edgesand a dense elastic infiltrate at the base.The ulcer surface is of red colour, painlesson palpation. What is the most likely di-agnosis?

A. Primary syphilisB. Primary tuberculosisC. Secondary tuberculosisD. CancerE. Secondary syphilis

55. A 32-year-old patient complains ofacute spontaneous attacks of pain in the14 tooth. The pain lasts for 10-20 minutesand occurs every 2-3 hours. Carious cavi-ty in the 14 tooth is filled with softeneddentin. Probing of the cavity floor is pai-nful at one point. Cold stimulus causespain. What is the most likely diagnosis?

A. Acute localized pulpitisB. Acute deep cariesC. Hyperemia of the pulpD. Exacerbation of chronic pulpitisE. Acute diffuse pulpitis

56. A 42-year-old patient complains of apainful ulcer in the mouth that is gettingbigger and does not heal over 1,5 months.Objectively: on the buccal mucosa there isa shallow soft ulcer 2 cm in diameter wi-th irregular undermined edges. The ulcerfloor is uneven and covered with yellow-gray coating. The ulcer is surrounded bymany small yellowish tubercles. Regionallymph nodes are elastic, painful, mattedtogether. Which disease is characterizedby such symptoms?

A. TuberculosisB. SyphilisC. Lichen planusD. CancerE. Ulcerative necrotizing stomatitis

57. A 24-year-old patient complains ofaching pain in the 11 tooth that is getti-ng worse during biting down on food.Two days ago the tooth was filled forpulpitis. Objectively: the 11 tooth is fi-lled. The thermal test causes no pain,vertical percussion is slightly painful. X-ray picture of the 11 tooth shows that theendodontic filling is 1 mm above the rootapex. Which of the following methods wi-ll be most effective for eliminating thiscomplication?

A. FluctuorizationB. Ultrahigh frequency therapyC. Relaxing incisionD. Submucous injection of 1% solution ofhydrocortisoneE. Analgetics

58. A patient is 12 years old. He hasbeen undergoing orthodontic treatmentfor pseudo prognathism with Angle’s fi-xed appliance for 10 months. What is theoptimal duration of the retentive period?

A. 20 monthsB. 10 monthsC. 6 monthsD. 3 monthsE. 12 months

59. A 55-year-old patient complains ofgeneral weakness, spasmodic neuralgicpain in the right side of face, rash inthe mouth and on the skin. Objectively:lip and chin skin is markedly hyperemic,there are numerous vesicles with clearexudate on the right. The right cheekmucosa is hyperemic, there is a string oferosions covered with fibrinous pellicle.What is the provisional diagnosis?

A. Herpes zosterB. Acute herpetic stomatitisC. Pemphigus vulgarisD. Allergic stomatitisE. Aphthous fever

60. According to the mother, a 5-year-old child complains about pain duringswallowing, weakness, body temperaturerise upt to 39, 5oC, swelling of submentallymph nodes. Objectively: the child’scondition is grave, body temperatureis 38, 8oC. Mucous membrane of theoral cavity is markedly hyperaemic and

Page 9: Krok 2 - 2013 Question Paper (Stomatology)

Krok 2 Stomatology 2013 9

edematic with haemorrhages and ulcerati-ons. Pharynx is markedly hyperemic,lacunae are enlarged and have necrosisareas. Regional, cervical, occipital lymphnodes are painful, enlarged and dense.What is the most likely diagnosis?

A. Infectious mononucleosisB. Acute herpetic stomatitisC. Necrotizing ulcerative gingivostomatitisD. Herpetic anginaE. Lacunar tonsillitis

61. A 19-year-old male patient consulteda dentist about gingival pain and bleedi-ng when eating and brushing his teeth.Objectively: anterior maxillary andmandibular teeth are overcrowded, thereis hyperemia, cyanosis and overgrowth ofgingival papillae up to 1/3 of the crownheight. The soft plaque is present. Thegums bleed when touched with a tool.What is the most likely diagnosis?

A. Chronic hypertrophic gingivitisB. Chronic catarrhal gingivitisC. Generalized periodontitisD. Localized periodontitisE. Ulcerative gingivitis

62. A 23-year-old female patient complai-ns of short-term pain caused by thermalstimuli in a lower jaw tooth on the ri-ght. Objectively: on the occlusal surfaceof the 46 tooth there is a deep cavity wi-th softened light dentin on the floor andwalls. Probing causes a slight pain acrossthe cavity floor. Pulp electroexcitabilityis 10 mA. During the cavity preparationa drop of blood showed up on its floor.What paste should be applied to the cavi-ty floor?

A. CalcidontB. CorticosteroidC. ArsenousD. ParaformaldehydeE. Resorcinol-formalin

63. After the clinical and radiographicexamination a 32-year-old patient was di-agnosed with chronic odontogenic sinusi-tis, fistula between sinus and oral cavitythrough the socket of the 27 tooth. Specifythe optimal method of surgical treatmentof this disease:

A. Radical maxillary sinusotomy and localtissue plasty of fistulaB. Local tissue plasty of fistula, anti-inflammatory therapyC. Radical maxillary sinusotomyD. Fistula plasty with a hard palate graftE. Fistula suturing, sinus lavage

64. A 15-month-old child diagnosed withintrusive luxation of the 51 tooth has beenreferred to a children’s dental surgeon.What is the optimal treatment tactics?

A. Monitoring the independent eruptionof the affected toothB. 51 tooth extractionC. 51 tooth repositionD. 51 tooth splintingE. 51 tooth replantation

65. A 45-year-old patient complains ofmissing teeth in the lower jaw on the ri-ght. Objectively: the 46, 45, 38, 48 teethare missing. The 47 tooth is mobile (1grade mobility), the crown of the 44tooth is decayed by 1/2. What orthopedicconstruction should be recommended forrestoration of masticatory efficiency andprevention of periodontal overload of the47, 44 teeth?

A. Bugel denture with clasp fixation forthe 47, 44, 34 teethB. Soldered bridge supported by the 48, 44teethC. Porcelain-fused-to-metal bridgesupported by the 47, 46 teethD. Bridge supported by the 47 toothE. Small saddle denture with clasp fixationfor the 47, 44 teeth

66. A 57-year-old patient complains oftooth mobility, inability to eat. Objecti-vely: the lower 35, 36, 37, 38, 44, 45, 46and 48 teeth are missing; the 31, 32, 33, 34,41, 42, 43, 47 teeth exhibit II grade mobi-lity, their clinical crowns are low, toothequator is not pronounced. What is theoptimal denture construction in this case?

A. Removable cast splintB. Removable partial dentureC. Kurlyandsky splint barD. Removable Bynin splintE. Removable splint with vestibulo-oralclasp

67. A child has been admitted to a hospitalwith a maxillofacial trauma. It is probablethat the wound has been contaminatedwith tetanus infection. How soon after theinjury should the tetanus serum be given?

Page 10: Krok 2 - 2013 Question Paper (Stomatology)

Krok 2 Stomatology 2013 10

A. Immediately at the time of seekingmedical helpB. Within 12 hoursC. Within 24 hoursD. Within 2 daysE. Within a week

68. A patient working as a bricklayercomplains of itching, burning, sorenessof lips that show up only in the summerperiod. He has a 3-year history of this di-sease. Objectively: vermilion border of thelower lip is hyperemic, edematic, coveredwith blisters and painful erosions 2 mm indiameter, crusts, cracks. What is the mostlikely diagnosis?

A. Actinic cheilitis, exudative formB. Contact allergic cheilitisC. Meteorological cheilitisD. Eczematous cheilitis, exudative formE. Exfoliative cheilitis, exudative form

69. Parents of a 2-month-old babycomplain about caseous films on themucous membrane of his cheeks andtongue. After examination the childhas been diagnosed with acute oralcandidiasis of moderate severity. Topi-cal treatment should be started with theadministration of the following agents:

A. AntifungalB. KeratoplasticC. AntisepticD. EnzymaticE. Antiviral

70. A patient complains of acute short-term toothache caused by thermal andchemical irritants. The tooth has not beentreated before, the pain arose about amonth ago. Objectively: on the occlusalsurface of the 36 tooth there is a deepcarious cavity within circumpulpal dentinwith overhanging chalky enamel. Probi-ng of the cavity floor causes a slight pain.There is also a short-term pain from thecold stimulus. The result of electric pulptest is 8 mA. What is the most likely di-agnosis?

A. Acute deep cariesB. Acute median cariesC. Pulp hyperemiaD. Chronic deep cariesE. Chronic fibrous pulpitis

71. A 20-year-old male patient complai-ns of acute pain during eating, badbreath, general weakness. Objectively:interdental papillae and marginal gingi-va are covered with dirty gray film, any

touch is painful. The body temperature isof 37, 5oC. Blood count: RBC - 4, 8 · 1012/l,Hb- 150 g/l, colour index - 0,9, WBC -10, 5 · 109/l, JG - 2%, BASO - 1%, EOS -5%, segmented neut. - 8%, stabs - 47%,LYM - 35%, PLT - 250 · 109/l. ESR - 20mm/h. What is the most likely diagnosis?

A. Vincent’s gingivitisB. AgranulocytosisC. Acute leukemiaD. Acute herpetic stomatitisE. Chronic leukemia

72. A 19-year-old patient complains ofindisposition, fever, pain in the gumsand oral mucosa. These presentationsshowed up two days ago. Objectively:body temperature is of 38,8 0C, regi-onal lymph nodes on the right areenlarged, painful, mobile. Oral mucosa ishyperemic, edematous; gingival papillaeare hemorrhagic, with grayish-white coati-ng, on the buccal mucosa there are painfululcers with irregular edges covered wi-th the same grayish-white coating. Whatorganisms are most likely to be found onmicrobiological study?

A. Fusospirillary symbiosisB. VirusC. Fungal pathogensD. Colon bacillusE. Diphtheria bacillus

73. A 30-year-old patient has a cut woundof the upper lateral surface of tongue. Thewound was given with a separating disk.Bood penetrated into the airways causingasphyxia. What form of asphyxia occurredin the patient?

A. AspirationB. StenoticC. ValvularD. ObstructiveE. Dislocation

74. A 10-year-old girl complains of a cari-ous cavity in a lower jaw tooth. Objecti-vely: the 26 tooth has a carious cavity wi-thin circumpulpal dentin, dentin is dense,pigmented, there is pain reaction to thecold stimulus, probing and percussionare painless. What is the provisional di-agnosis?

A. Chronic deep cariesB. Acute median cariesC. Acute deep cariesD. Chronic median cariesE. Chronic simple pulpitis

Page 11: Krok 2 - 2013 Question Paper (Stomatology)

Krok 2 Stomatology 2013 11

75. A 12-year-old girl visited a dentist fora regular check-up. Objectively: the teethare intact, the bite is open, the mucosain the papillary and marginal regions ishyperemic and cyanotic, the gingival papi-llae are roundish, enlarged and cover onethird of the crown height of the upperanterior teeth. Make a provisional di-agnosis:

A. Hypertrophic gingivitisB. Chronic catarrhal gingivitisC. Acute catarrhal gingivitisD. Gingival fibromatosisE. -

76. Parents of a 6-year-old child consulteda doctor about a swelling in his rightparotid-masseteric region, dry mouth,fever up to 37, 7oC. Similar presentationsturned up about 6 months ago. Examinati-on revealed a tuberous, slightly painfulinfiltrate in the right parotid-massetericregion. Saliva is viscous, the right salivarygland duct discharges some secretion withwhitish inclusions. What is the most likelydiagnosis?

A. Exacerbation of chronicparenchymatous parotitisB. Acute purulent parotitisC. Mixed tumor, salivary gland typeD. Acute purulent lymphadenitis ofparotid glandE. Parotitis

77. A 34-year-old patient was undergoingtreatment for acute deep caries of the 37tooth. During the tooth preparation thebuccal pulp horn was accidentally opened.What is the most rational treatment of the37 tooth?

A. Biological methodB. Vital amputationC. Non-vital amputationD. Vital extirpationE. Non-vital extirpation

78. A 21-year-old patient complains of aconstant progressing throbbing pain in the27 tooth. Objectively: a large carious cavi-ty is filled with softened dentin, the toothcavity is closed. Probing of the cavity flooris painless, percussion causes acute pain,there is grade II tooth mobility. Palpationof the mucous membrane in the projecti-on of the root apex of the 27 tooth causespain. Radiological changes are absent.What is the most likely diagnosis?

A. Acute purulent periodontitisB. Exacerbation of chronic periodontitisC. Acute diffuse pulpitisD. Acute serous periodontitisE. Acute purulent pulpitis

79. A 20-year-old patient complains of the23 tooth sensitivity to sour or sweet food.The pain abates after the irritants areremoved. The problems arose two weeksago. Objectively: on the vestibular surfaceof the 23 tooth there are white spots 2-3mm in diameter. The spots have a roughsurface, methylene blue stain cannot bewashed away with water. What is the mostlikely diagnosis?

A. Acute initial cariesB. Chronic initial cariesC. Mottled enamelD. Initial stage of enamel necrosisE. Local enamel hypoplasia

80. A 24-year-old patient complains ofacute pain and swelling in the submandi-bular triangle that is getting worse whenhe eats spicy food. He has been diagnosedwith exacerbation of chronic calculous si-aloadenitis with the concrement localizedin the median part of the salivary duct.Which method is most rational in thiscase?

A. Duct dissection and concrementremovalB. Gland extirpationC. Duct bougienageD. Ptyalalogue dietE. Physiotherapy

81. A 38-year-old patient with chronicgeneralized periodontitis has beenreferred to orthopedic treatment. Objecti-vely: dentitions are without gaps, the 12,11, 21, 22 teeth are pulpless and exhibit Igrade mobility. The other teeth are stable.What is the most aesthetic splint for theanterior teeth?

A. Mamlok splintB. Ring adhesion splintC. Soldered combined crownsD. Cap splintE. Brace

82. A 42-year-old patient complains ofpain in the right side of her head, restri-cted movements of the lower jaw, cli-cking sound, periodic spasms of chewi-ng muscles. Objectively: the face issymmetric, mouth opening is restricted.On palpation of the right temporo-mandibular joint (TMJ) there are crepi-

Page 12: Krok 2 - 2013 Question Paper (Stomatology)

Krok 2 Stomatology 2013 12

tation and clicking accompanying mandi-ble movements. Examination of the oralcavity revealed also a Kennedy’s class IIdefect on the right. What is the most likelydiagnosis?

A. Pain dysfunction of the right TMJB. Acute arthritis of the right TMJC. Sclerosing osteoarthritis of the rightTMJD. Contracture of the right TMJE. Myositis ossificans

83. A patient needs a clasp prosthesis. It isplanned to study the jaw model by meansof a parallelometer in order to determinethe required depth of the undercuts on theabutment teeth. Specify the length of themeasuring rods used for this purpose:

A. 0,25 0,50 0,75B. 0,15 0,40 0,65C. 0,20 0,45 0,70D. 0,30 0,55 0,80E. 0,35 0,60 0,85

84. An 18-year-old male has a furunclein the region of mandible angle. Thesurrounding tissues are rigid. Cyanosisand induration in form of a cord extendto the corner of the eye. General conditi-on is deteriorated, body temperature is of38, 5oC. What complication has developedin this case?

A. Facial vein thrombophlebitisB. Abscess of cheekC. Phlegmon of cheekD. Acute sinusitisE. Carbuncle of cheek

85. A 30-year-old patient needs a post andcore for the 36 tooth prior to the toothrestoration with a crown. Objectively: thecrown of the 36 tooth is decayed downto the gums, percussion is painless. Radi-ograph shows that root canals are filled tothe top. There are no changes of the peri-apical tissues. What is the optimal numberof posts in this case?

A. TwoB. ThreeC. FourD. OneE. Five

86. A 25-year-old patient presented toa clinic on the fourth day after a facetrauma complaining of pain in the lowerjaw on both sides, difficult swallowing.Objectvely: there is a slight swelling in themental region, dentitions are intact but a

fragment of 44 43 42 41 31 32 33 34 teethis displaced downwards and backwards.Single-step fragment reposition is noteffective. Which appliance should be usedfor the fragment reposition?

A. Post’sB. Kurlyandsky cap splint for one jawC. Weber’sD. Betelman’sE. Schur’s

87. A 25-year-old patient has necrosis focialong the gingival margin on both jaws.Face skin is sallow, he looks older than hisage. Submandibular, mental, cervical andaxillary lymph nodes are enlarged, pai-nless, mobile. The patient leads a hecticlife, abuses drugs. Over the last monthhe has complained of weakness, constantbody temperature of 37, 3 − 37, 5oC, di-arrhea, weight loss of 15 kg. What is themost likely provisional diagnosis?

A. AIDSB. Acute leukemiaC. Hypovitaminosis CD. Vincent’s gingivitisE. Agranulocytosis

88. A 20-year-old patient was delivered tothe hospital 6 hours after injury. Objecti-vely: the patient has malocclusion due tothe displacement of bone fragments. Theteeth on both jaws are intact except for themissing 35 tooth. Ro-gram shows fractureof mandible. The fragments are displaced.What treatment is most appropriate?

A. Anchor splints with intermaxillaryrubber traction bandB. Intraoral miniplate osteosynthesisC. Smooth braceD. Intermaxillary ligatureE. Chin cup

89. A 6-year-old girl has unrestrictedopening of the mouth. The alveolarprocess of the upper jaw is intact. Soft andhard palate are of a triangular shape andhave a cleft up to the level of the 13 and23 teeth. Soft palate is shortened. Speechis indistinct. The child was born with thisdefect. What is the most likely diagnosis?

A. Natural partial cleft palateB. Isolated complete cleft palateC. Submucous cleft palateD. Isolated partial uranoschisisE. -

90. 3 days ago a 35-year-old patientunderwent the extraction of the 47 tooth,

Page 13: Krok 2 - 2013 Question Paper (Stomatology)

Krok 2 Stomatology 2013 13

which was followed by health deteriorati-on. Objectively: the patient is pale, bodytemperature is of 38, 1oC, the lower jawon the left is thickened, the surroundi-ng soft tissues are swollen, opening ofthe mouth is difficult. Mucogingival foldat the 48, 47, 46 teeth is edematous,mucous membrane is hyperemic. Thereare purulent discharges from the 47 toothsocket, percussion of the 48, 46, 45 teethis painful. There is also paresthesia in thelower lip region. What is the most likelydiagnosis?

A. Acute mandibular osteomyelitisB. Acute mandibular periostitisC. Submandibular phlegmonD. Acute periodontitis of the 47 toothE. Abscess of mandibular radicular cyst

91. Parents of a 6,5-year-old boy consultedan orthodontist about no contact betweenthe front teeth. The child has a badhabit of sucking his tongue. Objectively:there is a symptom of multiple pits in hischin when the lips are closed, speech di-sturbance, between the front teeth thereis a vertical gap up to 8 mm. Specify theocclusion anomaly:

A. Open biteB. Cross-biteC. Distal occlusionD. Mesial biteE. Overbite

92. During the extraction of the 17 tooththe distal buccal root broke off and remai-ned in the socket. Select a tool to extractthe root:

A. Bayonet forceps, straight elevatorB. Straight forceps, straight elevatorC. S-shaped forcepsD. Beak-shaped forceps, angular elevatorE. Angular elevators

93. A 64-year-old patient needs acomplete removable denture for theupper jaw. During fitting the Herbst appli-ance, the orthopedist revealed that itfalls off when the patient swallows. Thecorrection is required in the followingarea:

A. Distally along the "A"lineB. In the region of maxillary prominencesC. In the region of buccal musclesD. Vestibularly between caninesE. Along the vestibular edge

94. A 5-year-old child complains of acutespontaneous pain in an upper jaw tooth

on the right. Hot stimuli make the painsignificantly worse. The tooth had beenpreviously treated for median caries. Thepain arose a day before. On the medialocclusal surface of the 54 tooth there isa deep carious cavity, the tooth cavity isclosed. Probing of the cavity floor is pai-nless, percussion is slightly painful. Afternecrotomy the pulp horn was opened.Deep probing is painful. What is theprovisional diagnosis?

A. Acute purulent pulpitisB. Acute serous pulpitisC. Exacerbation of chronic pulpitisD. Acute purulent periodontitisE. Exacerbation of chronic periodontitis

95. A 7-year-old child complains of painand swelling in the left submandibularregion. The swelling in this region showedup 2 days ago. Objectively: the child is insatisfactory condition, body temperatureis of 37, 3oC. Face is asymmetrical dueto the soft tissue swelling in the leftsubmandibular region. Palpation revealsa round formation sized 2x2 cm. Theformation is mobile, painful, not attachedto skin. The 74 tooth is changed in colour,percussion is painful. What is the provisi-onal diagnosis?

A. Acute serous odontogenic lymphadeni-tis of the left submandibular regionB. Acute serous nonodontogeniclymphadenitis of the left submandi-bular regionC. Acute purulent odontogeniclymphadenitis of the left submandi-bular regionD. Phlegmonous adenitis of the rightsubmandibular regionE. Lateral cervical cyst

96. A 21-year-old patient consulted adental surgeon about a tumour-likeformation on the left cheek. The pati-ent noticed the swelling for the first ti-me a year ago. Objectively: on the leftcheek skin there is a painless roundi-sh pedunculated tumour up to 0,5 cmin diameter. The surface of the formati-on looks like cauliflower, the surroundingskin is not changed. What is the most li-kely diagnosis?

A. Papilloma of cheekB. Cutaneous hornC. Hemangioma of cheekD. Botryomycoma of cheekE. Fibroma of cheek

97. A 24-year-old patient complains of a

Page 14: Krok 2 - 2013 Question Paper (Stomatology)

Krok 2 Stomatology 2013 14

growing tumour in the body of mandibleon the right. The tumour growth has beenobserved for 5 months. Objectively: inthe angle of mandible there is a swelling.Regional lymph nodes are not enlarged.Ro-gram shows a well-defined focus ofbone destruction at the level of the 46, 47,and 48 teeth and in the ramus of mandible.The lesion is in form of individual cavitieswith osseous septa. The roots of the 46, 47,48 teeth are resorbed. What is the provisi-onal diagnosis?

A. Osteoclastoma of mandible on the rightB. Ameloblastoma of mandible on therightC. Follicular cyst of mandible on the rightD. Radicular cyst of mandible on the rightE. Odontoma of mandible on the right

98. Parents of a 6-year-old girl consultedan orthodontist about protrusion of thelower jaw. The child looks like his father.Objectively: the child has primary bi-te, there are diastemata and tremata onboth jaws, reverse incisal overlap of frontteeth, the sagittal gap is up to 3 mm, thelateral parts are characterized by mesio-occlusion. Ilyina-Markosyan test for thedistal displacement of mandible is negati-ve. What principle of treatment will bemost effective?

A. To delay the growth of mandible insagittal directionB. To stimulate the growth of maxilla insagittal directionC. The treatment is not requiredD. No to start treatment until the end oftransitional dentitionE. To start orthodontic treatment after thesecond dentition is completed

99. A 52-year-old patient complains ofpain and a swelling in the right parotidregion. These manifestations have beenpresent for about 2 years. Over the lastmonth the swelling has grown bigger, painhas intensified. Objectively: the face isasymmetric due to the dense infiltratein the right parotid region. The poorlycircumscribed, painful formation infi-ltrates the surrounding tissues. At theright side of neck in front and behindthe sternocleidomastoid muscle there areenlarged, dense, mobile lymph nodes.The right naso-buccal groove is flattened,the corner of mouth is downturned. Themouth opens freely. The are pronouncedsymptoms of the right facial nerve paresis.What disease can be suspected?

A. Adenocarcinoma of the parotid salivaryglandB. Chronic parotitisC. Actinomycosis of the parotid-masseterregionD. Chronic lymphadenitisE. Pleomorphic adenoma of the parotidgland

100. A 58-year-old patient complains ofmissing teeth on the upper jaw. Objecti-vely: in the place of the missing centralupper incisors there is a fan-shaped vesti-bularly located cord on the alveolarprocess. The cord is 1 cm wide, it makesprosthetics more complicated. What is theoptimal medical tactics?

A. Surgical excision of the cordB. Silicone implantC. Operation on the alveolar processD. Not to cover the cord with the dentureE. To cover the cord with the denture

101. A 35-year-old patient has been di-agnosed with chronic median caries ofthe 36 tooth. There is a Black’s class IIcavity affecting masticatory surface. Whatmaterial should be chosen for the toothfilling?

A. Light-cure microhybride compositeB. Glass ionomer cementC. Silicophosphate cementD. Light-cure fluid compositeE. Light-cure microfilled composite

102. A 28-year-old patient complains ofdull pain in the 17 tooth caused by hotmeal, the pain slowly abates after removi-ng the irritant. There is also a feeling oftooth bursting, and bad breath after thetooth suction. What is the provisional di-agnosis?

A. Chronic gangrenous pulpitisB. Chronic fibrous pulpitisC. Chronic fibrous periodontitisD. Chronic hypertrophic pulpitisE. Chronic concrementous pulpitis

103. Parents of an 8-year-old child withDown syndrome took the child to a doctorfor oral cavity sanitation. After the exami-nation entailing great difficulties the childwas found to have four teeth decayed as aresult of chronic periodontitis. What kindof anesthesia should be chosen for surgi-cal sanitation in one visit?

Page 15: Krok 2 - 2013 Question Paper (Stomatology)

Krok 2 Stomatology 2013 15

A. PhlebonarcosisB. Conduction anesthesiaC. Mask anesthesiaD. Endotracheal anesthesiaE. Endotracheal anesthesia through atracheostoma

104. A patronage nurse visited a newbornbaby. Examination revealed the shortenedlower part of the face, the backward-sloping chin, missing teeth, the retroposedlower jaw. What is the number of dentalfollicles in each jaw of a newborn baby?

A. 18B. 16C. 14D. 12E. 10

105. A 47-year-old patient complains of aburning sensation and pain in the mouth.Objectively: on the mucous membrane ofcheeks along the line of teeth contact andin the corners of mouth there are multiplepolygonal bright red erosions 1,0-1,5 cmin diameter located on the hyperkeratini-zed plaque and opaque whitish mucosa.Cytological analysis revealed keratinizingepithelial cells. What is the most likely di-agnosis?

A. Leukoplakia, erosive formB. Lichen planus, erosive formC. Erythema multiformeD. Secondary syphilisE. Lupus erythematosus, erosive form

106. A 67-year-old female patient has aburning sensation under her completeremovable denture for the upper jawwhich was fabricated of plastics 3 daysago. Examination revealed hyperemia andedema of the mucous membrane withinthe boundaries of the denture-supportingarea. What is the cause of this pathology?

A. Excess of monomer in the plasticsB. Sensitization to the plasticsC. Paresthesia of the oral mucosaD. Dysbiosis of the oral cavityE. Galvanism in the oral cavity

107. A 55-year-old patient consulted adentist about a roundish tumour-likeformation of about 1 cm in diameterlocated within the vermilion border ofhis lower lip. Objectively: the tumour-like formation protrudes about 5 mmabove the vermilion border, it is dense andgrayish-red. The surface of the formationis covered with thin scales that can hardlybe removed. What is the most likely di-

agnosis?

A. Verrucous precancer of the vermilionborder of lipB. Abrasive precancerous Manganotti’scheilitisC. Precancerous limited hyperkeratosis ofthe vermilion border of lipD. Bowen’s diseaseE. Erythroplasia of Queyrat

108. A 16-year-old male patient complai-ns of itchy rash on the vermilion border oflips and in the mouth corners. The patienthas a 4-year-old history of this disease wi-th exacerbation periods mainly in autumnand winter. Objectively: vermilion borderof lips and skin in the mouth corners is dry,peeling with small scales. Mouth cornersare characterized by lichenification, deepfolds and fissures, bloody crusts. What isthe most likely diagnosis?

A. Atopic cheilitisB. Actinic cheilitisC. Meteorological cheilitisD. Eczematous cheilitisE. Exfoliative cheilitis

109. Preventive examination of a 28-year-old male patient revealed reddish smoothpatches in form of different-sized circleson the back of tongue. The patches arecircumscribed with a narrow whitish stri-pe. It is known that these tongue changesare not permanent, show up and di-sappear throughout a few years and don’tbother him. The patient has a history ofchronic gastritis. Histological examinati-on of the epithelium revealed hyper- andparakeratosis. What is the most likely di-agnosis?

A. Desquamative glossitisB. Secondary syphilisC. Lichen planusD. LeukoplakiaE. Rhomboid glossitis

110. After unilateral resection of theupper jaw a 52-year-old patient receivedthe immediate-insertion denture. What isthe term of its use?

A. 1-3 monthsB. 1-2 monthsC. 7-12 monthsD. 2-3 yearsE. 4-5 years

111. An injured patient complains of limi-ted opening of the mouth, nose bleedi-ng, skin numbness in the infraorbital and

Page 16: Krok 2 - 2013 Question Paper (Stomatology)

Krok 2 Stomatology 2013 16

lower eyelid region. Objectively: there isface deformation due to the depression ofsoft tissues in the left cheekbone region,step deformity in the middle part of theinferior margin of the left orbit and in thearea of the zygomatic alveolar crest. Whatis the most likely diagnosis?

A. Zygomatic bone fracture with di-splacement of bone fragmentsB. Fracture of the right zygomatic bonewithout displacement of fragmentsC. Le Fort I fracture of maxillaD. Le Fort II fracture of maxillaE. Fracture of the malar arch

112. A 30-year-old patient needs tohave his 26 tooth extracted because ofexacerbation of chronic periodontitis.Objectively: the crown of the 26 tooth isdecayed by 1/3. What forceps can be usedfor this tooth extraction?

A. S-shaped forceps with a projecting tipon the left beakB. S-shaped forceps with a projecting tipon the right beakC. Straight forcepsD. Straight elevatorE. S-shaped forceps without projecting tips

113. A patient is 56 years old. 6 monthsafter resection of the right side of theupper jaw the patient is going to receivea permanent resection prosthesis. Whatdevice can be used to determine theoptimal state of the resection prosthesisborders?

A. ParallelometerB. MicrometerC. Larin’s applianceD. GnathodynamometerE. Myotonometer

114. A 49-year-old patient consulted adental surgeon about the oral cavitysanitation. He has an indication for theextraction of the 16 tooth. History: thetooth hasn’t been treated before, it hasdecayed within the last 4 years. Objecti-vely: the 16 tooth’s crown is decayedby over 2/3, the mucosa exhibits nopathological changes. Which tool is requi-red for the tooth extraction?

A. Bayonet root forcepsB. S-shaped forceps (right)C. Straight elevatorD. S-shaped closed-beak forcepsE. Crown bayonet forceps

115. A patient complains of limited

mouth opening, pain during swallowi-ng, fever up to 38, 5oC, weakness, indi-sposition. Objectively: the mouth opensup to 1 cm. After Berchet anesthesiaexamination of the oral cavity revealededema, hyperemia, tenderness of thepterygomandibular fold. Lateral andposterior pharyngeal walls are intact. The38 tooth is semi-impacted. Palpation ofthe internal surface of the mandibularangle is painful. What is the most likelydiagnosis?

A. Phlegmon of pterygomandibular spaceB. Acute tonsillitisC. Phlegmon of the infratemporal regionD. Acute pericoronitis of the 38 toothregionE. Phlegmon of peripharyngeal space

116. A 6-year-old patient complains of along-lasting pain in a lower right tooththat is getting worse when biting downon food. The tooth has been previouslytreated for caries. Objectively: the cavi-ty on the occlusal surface of the 36 toothis filled, percussion is painful. Along themucogingival fold in the projection ofthe 36 tooth roots there is a fistula withpurulent discharge. What is the most li-kely diagnosis?

A. Chronic granulating periodontitisB. Exacerbation of chronic periodontitisC. Chronic fibrous periodontitisD. Chronic granulomatous periodontitisE. Radicular cyst of mandible

117. A 35-year-old patient complains of aneoplasm on the tip of his tongue whichhe injures with his teeth. The neoplasmsometimes increases, and sometimesdecreases in size. Objectively: on the tipof tongue there is a roundish well-definedneoplasm 0,5 cm in diameter with broadbase. The neoplasm is the same colour asthe mucosa of tongue. What is the mostlikely diagnosis?

A. Papilloma of tongueB. Abscess of tongueC. Lipoma of tongueD. Hemangiofibroma of tongueE. Fibroma of tongue

118. A patient presented to a doctor witha red spot on her chin skin. Two monthsago the patient suffered a chin injury.After resorption of traumatic edema thereappeared a black punctum which wasgradually increasing in size. Objectively:the face is symmetric, on the chin skinthere is an irregular wine-coloured pai-

Page 17: Krok 2 - 2013 Question Paper (Stomatology)

Krok 2 Stomatology 2013 17

nless spot sized 2x1,5 cm that turns whitewhen pressed. What is the most likely di-agnosis?

A. Capillary hemangiomaB. Pigmented nevusC. MelanomaD. HematomaE. Telangiectasia

119. X-ray examination of the oral cavi-ty revealed a pronounced damage of thecompact bone, mild osteoporosis of apicesof interalveolar septa and widened peri-odontal ligament space around the dentalnecks. What is the most likely diagnosis?

A. Chronic generalized periodontitis,initial stageB. Chronic generalized periodontitis, IstageC. Chronic catarrhal gingivitisD. Periodontosis, I stageE. Periodontosis, initial stage

120. Preventive examination of a 13,5-year-old child revealed congestivehyperemia and a slight edema of the gi-ngival margin in the region of the frontteeth on both jaws. The patient has peri-odic gingival haemorrhages during toothbrushing. DMF index is 2. What kind oftoothpaste should be recommended forthe individual oral hygiene?

A. Toothpaste containing herbal extractsB. Toothpaste containing calciumC. Toothpaste containing aminofluoridesD. Toothpaste containing sodium fluorideE. Toothpaste containing salt additives

121. A 64-year-old patient withedentulous jaws has a minor uniformatrophy of maxilla and a pronouncedatrophy of lateral parts of mandible. Themucosa in these parts makes longitudi-nal folds, the alveolar crest is mobile inthe anterior part. The condition of mandi-ble should be taken into considerationat the following stage of fabrication of acomplete removable denture:

A. Taking differentiated functionalimpressionB. Taking anatomical impressionC. Taking compression functional impressi-onD. Taking decompression functionalimpressionE. Determining the central occlusion

122. A 24-year-old female patientconsulted a dentist about pain in the 26

tooth. After a physical examination thepatient was diagnosed with exacerbati-on of chronic periodontitis of the 26tooth. During the tooth extraction thecoronal part of the tooth was acci-dentally broken. Further manipulationswith bayonet forceps failed to extract thetooth roots. What actions are to be takenfor the successful root extraction?

A. To disjoin the roots by using a drill andfissure burrB. To send the patient to the hospitalC. To use Lecluse elevatorD. To complete the tooth extraction duringthe next visitE. To use angular elevator

123. A 30-year-old female patientcomplains of limited mouth opening. Shehas a history of a trauma received at thetime when her jaws were closed. Objecti-vely: mouth opening is limited to 1,5cm, the bite is normal, mouth openingcauses the displacement of mandible tothe left. Palpation reveals pain in the lefttemporomandibular joint (TMJ). X-raypicture shows that the bone integrity ispreserved. What is the most likely di-agnosis?

A. Acute traumatic arthritis of the leftTMJB. Fracture of the left articular process ofmandibleC. Acute unilateral dislocation of the TMJD. Contusion of the left TMJE. Fracture of the left cheekbone withdisplacement of bone fragments

124. Preventive examination of a 5-year-old child revealed a habit of lower lip biti-ng. What malocclusion may develop if thechild keeps this habit?

A. Anterior biteB. Prognathic biteC. Open biteD. Deep overbiteE. Cross-bite

125. A 14-year-old girl complains of indi-stinct pronunciation that developed at theage of 14 after the acute respiratory vi-ral disease. Examination revealed normalface and normal teeth alignment, occlusaldisharmony was not found. Palpation di-dn’t reveal cleft palate. Uvula doesn’tmove during pronunciation of sounds, itspalpation does not cause gag reflex. Whatis the reason for indistinct pronunciationof sounds?

Page 18: Krok 2 - 2013 Question Paper (Stomatology)

Krok 2 Stomatology 2013 18

A. Paresis of the soft palate and uvulamusclesB. Adenoid vegetationsC. Palatal slitD. Hypertrophy of lingual tonsilE. Deformation of the bite

126. A 35-year-old female patient seeksprosthetic dentistry. Objectively: the 18,14, 13, 12, 11, 21, 22, 23, 24, 28 teethneed restoration. The crowns confiningthe defect are high and stable. To restorethe integrity of the dentition it is plannedto fabricate a cast clasp denture. What ki-nd of mechanical fixator should be used?

A. Bar fixationB. Supporting-retaining claspsC. Retaining claspsD. Telescopic fixationE. Dentoalveolar clasps

127. A 22-year-old patient presented to aprosthetic dentistry clinic because of mi-ssing of the 21 tooth, the 11 and 22 teethare intact. The 21 tooth was extracted 2months ago. What construction is mostsuitable in this case?

A. Two-stage implantationB. One-stage implantation with si-multaneous fabrication of porcelain-fused-to-metal crownC. Partial removable dentureD. Swaged and soldered denture supportedby the 11 and 22 teethE. Metal-plastic denture supported by the11 and 22 teeth

128. A 48-year-old patient has undergoneunilateral resection of the upper jaw. Heneeds a resection prosthesis. Objectively:the 21, 22, 23, 24, 25, 26 teeth are stable,hard palate is broad, flat. What connecti-on of clasps with the base will reduce theinstability of the resection prosthesis?

A. ArticularB. StableC. ElasticD. FixedE. Does not matter

129. A 47-year-old patient needs dentalprosthetics. Objectively: the 24, 25 teethare missing, there is a slight dento-alveolarlengthening in the region of the 34, 35teeth. Which of the following methods ofthe occlusal surface correction should beapplied?

A. Shortening of teethB. Increase of interalveolar heightC. Orthodontic methodD. Orthodontic and surgical methodE. Tooth extraction

130. It is planned to open a city dental cli-nic to serve 200,000 of adult population.How many positions of dentists should beprovided for the orthopedic department?

A. 20B. 30C. 15D. 10E. 5

131. A 25-year-old patient consulteda dentist about orthopedic treatmentof the 17 tooth. After the patienthad been given local anesthesia, hiscondition suddenly deteriorated (hedeveloped anxiety, general weakness,acute headache, nausea, loss of consci-ousness, poor pupillary reaction to light,abrupt drop of arterial pressure. What isthe most likely diagnosis?

A. Anaphylactic shockB. Cardiogenic shockC. Septic shockD. Traumatic shockE. Toxic shock

132. The department of maxillofacialsurgery admitted a patient who needsrepair of a post-traumatic defect of nosewing up to 3,0 cm in diameter. The traumaoccured six months ago. What kind ofgrafting is indicated in this clinical situati-on?

A. Grafting with chondrocutaneous flap ofthe auricleB. Grafting with local tissues of nasolabialor cheek regionsC. Grafting with pedicle flap of frontal andbuccal regionsD. Grafting with tubed pedicle (Filatov’s)flapE. Free grafting with dermal flap

133. A 25-year-old female patientconsulted a dentist about acute pain inthe upper jaw on the left. The pain occursduring eating. Objectively: on the distalapproximal surface of the 26 tooth there isa cavity filled with light soft dentin. Probi-ng causes a slight pain along the dentin-enamel junction, percussion is painless.Cold water causes quickly abating pain.What is the most likely diagnosis?

Page 19: Krok 2 - 2013 Question Paper (Stomatology)

Krok 2 Stomatology 2013 19

A. Acute median cariesB. Chronic median cariesC. Acute deep cariesD. Chronic fibrous pulpitisE. Chronic deep caries

134. At a prosthetics dentistry clinic a35-year-old patient received a porcelain-fused-to-metal crown for the 21 tooth.What is the minimum warranty period forporcelain-fused-to-metal crowns underthe current legislation?

A. 12 monthsB. 24 monthsC. 36 monthsD. 6 monthsE. 3 months

135. During the surgical removal of aretention cyst of the lower lip a 14-year-old boy complained of sudden weakness,dizziness, nausea. Objectively: the skin iscovered with cold sweat. Respiration isfrequent, pulse is weak, AP is decreased(90/60 mm Hg), the hands are cold. Whatis the most likely diagnosis?

A. SyncopeB. Toxic shockC. Traumatic shockD. Anaphylactic shockE. Collapse

136. Parents of a 7,5-year old child broughthim to a dentist for oral cavity sanitation.Objectively: DEF (for primary teeth) +DMF (for permanent teeth) index = 4,Green-Vermillion index = 2,5. Fissuresof the first permanent molars are open,intact, non-pigmented. What method ofprimary prevention of dental caries maybe appropriate in this case?

A. Non-invasive hermetizationB. Invasive hermetizationC. Application of fluoride varnishD. Application of antibacterial varnishE. Application of calcium gels

137. A 28-year-old patient complains ofexperiencing gum discomfort, gingivalhaemorrhages and pain, especially duringeating, for a week. Objectively: the gumsare swollen and markedly hyperemic,especially within the 43, 42, 41, 31, 32, 33teeth, soft plaque and tartar are present,gingival sulcus is up to 2 mm deep. Whatis the most likely diagnosis?

A. Acute deep gingivitisB. Chronic catarrhal gingivitisC. Hypertrophic gingivitisD. Initial stage of chronic generalizedperiodontitisE. Exacerbation of initial generalizedperiodontitis

138. A 35-year-old patient complains ofgingival hemorrhage during tooth brushi-ng. Objectively: gums of both jaws arehyperemic and cyanotic, supragingivaland subgingival tartar is present. The peri-odontal pockets are 4-6 mm deep. What isthe most likely provisional diagnosis?

A. II grade generalized periodontitis,chronic courseB. Catarrhal gingivitisC. Hypertrophic gingivitisD. Exacerbation of I grade generalizedperiodontitisE. II grade parodontosis

139. A 27-year-old male patient complai-ns of aching long-lasting pain in the 15tooth during having meals, especially coldfood. Sometimes the pain occurs when thetemperature changes. Objectively: on thedistal surface of the 15 tooth there is acavity filled with softened dentin. Probingis painful. Electroexcitability of the pulp is35 µA. What is the most likely diagnosis?

A. Chronic fibrous pulpitisB. Acute deep cariesC. Chronic deep cariesD. Hyperemia of the pulpE. Exacerbation of chronic pulpitis

140. A 12-year-old child presents withbody temperature of 38oC, chills, nausea,vomiting, delirium, weakness. In the mi-ddle third of face there is butterfly-shapedrash. Regional lymph nodes are enlarged,slightly painful. Blood count: WBC- 12 ·109/l, LYMP - 8, 0 · 109/l, ESR - 26 mm/h.Make a diagnosis:

A. ErysipelasB. Facial vein phlebitisC. Cutaneous actinomycosisD. Streptoderma. SubmandibularlymphadenitisE. Acute nonodontogenic sinusitis

141. A 37-year-old patient had an appoi-ntment with his orthopaedist about dentalprosthetics. During the preparation ofhard dental tissues for a fixed denture hewas given conduction anesthesia. Duringthe injection the patient developed coldsweat, sudden pallor and cyanosis, thready

Page 20: Krok 2 - 2013 Question Paper (Stomatology)

Krok 2 Stomatology 2013 20

pulse. The patient stayed conscious, butbecame relaxed and lethargic. What is theprovisional diagnosis?

A. CollapseB. Anaphylactic shockC. IntoxicationD. SyncopeE. Hypertensive crisis

142. A 25-year-old patient complains of alight brown spot in the upper front tooth.Objectively: the 23 tooth has a single li-ght brown spot in the precervical regi-on. Probing reveals smooth surface. Thetooth is nonresponsive to cold and probi-ng. What is the most likely diagnosis?

A. Chronic initial cariesB. FluorosisC. Local enamel hypoplasiaD. Acute initial cariesE. Chronic superficial caries

143. A 48-year-old patient is in gravecondition: body temperature is of 39, 8oC,the face is asymmetric due to the infiltrati-on of the submental and both submandi-bular triangles. Speech is difficult as thetongue is displaced upwards. Swallowingis painful. Sublingual plicae are markedlyhyperemic and edematous. What is themost likely diagnosis?

A. Phlegmon of mouth floorB. Abscess of tongue rootC. Phlegmon of submental triangleD. Phlegmon of alveololingual grooveE. Phlegmon of submandibular triangle

144. During the dental care procedures a4-year-old child has been found to havea carious cavity in the 64 tooth. The cavi-ty filled with softened dentin is locatedwithin parapulpal dentin and doesn’tcommunicate with the tooth cavity. Probi-ng of the cavity floor is painless. Gingivalmucosa in the projection of the root apicesof the 64 tooth exhibits a healed fistula.What is the most likely diagnosis?

A. Chronic granulating periodontitisB. Chronic periodontitis fibroticC. Chronic fibrous pulpitisD. Chronic gangrenous pulpitisE. Chronic periodontitis garnulematozny

145. A 20-year-old patient in the 8thmonth of pregnancy complains about anaesthetic defect, speech disturbance dueto the missing upper front tooth. Objecti-vely: the 12 tooth is missing. The 11,13 teeth are intact, stable, of anatomi-

cal shape. The patient has orthognathicbite. What prosthesic design should bepreferred?

A. Immediate prosthesisB. Plastic bridgeC. Porcelain-fused-to-metal bridgeD. Plastic-to-metal bridgeE. Implant with a pocelain crown

146. A 62-year-old patient diagnosed withTMJ arthritis presented to a clinic. Thepatient underwent radiography. Speci-fy the radiological presentations of TMJarthritis:

A. Changed shape of the bone elements ofthe jointB. Joint space narrowingC. Joint space wideningD. No joint spaceE. Compaction of the articular head cortex

147. A 24-year-old patient consulted adentist about a cavity in an upper jawtooth, which appeared three months ago.The tooth had not ached before. Objecti-vely: on the occlusal surface of the 15tooth there is a small carious cavity withinthe mantle dentin filled with leftover food.Probing revealed that the cavity walls andfloor were dense. The response to thermalstimuli is absent. What is the most likelydiagnosis?

A. Chronic median cariesB. Initial cariesC. Acute deep cariesD. Acute median cariesE. Pulp hyperemia

148. A 45-year-old patient has a bilateralmental mandibular fracture with a typicaldisplacement of fragments. What actionsshould be taken to eliminate asphyxia?

A. Pull out and pierce the tongueB. Artificial pulmonary ventilationC. Tracheal intubation through the noseD. Tracheal intubation through the mouthE. Tracheostomy

149. A 42-year-old patient consulted adentist about a swelling on the left cheek,redness of its skin. 2 months ago he noti-ced an induration on the left cheek whichhas been eventually enlarging. The skinover it turned bluish-red and soft in thecenter. Objectively: the left cheek skin hasa lesion sized 2,5x1, 5 cm with regularedges, the skin is dramatically thinned,cyanotic, glossy, makes folds and bumps.Intraoral palpation reveals a cord going

Page 21: Krok 2 - 2013 Question Paper (Stomatology)

Krok 2 Stomatology 2013 21

from the 37 tooth to the cheek lesion. The37 tooth is filled. What is the most likelydiagnosis?

A. Odontogenic facial granulomaB. Lymphadenitis of cheekC. Furuncle of cheekD. Phlegmon of cheekE. Carbuncle of cheek

150. A 40-year-old patient consulted adental surgeon about difficult mouthopening. Clinical examination revealedmouth opening limited to 1 cm. The 48tooth crown is decayed by 2/3, percussionis strongly positive. What kind of peri-pheral conduction anesthesia should beapplied for the surgical extraction of the48 tooth?

A. Subzygomatic Berchet-DubovanesthesiaB. Mandibular anesthesiaC. Torusal (Weisbrem’s) anesthesiaD. Extraoral mandibular anesthesiaE. Mental anesthesia

151. A 39-year-old patient had been admi-tted to the maxillofacial surgery unitfor a swelling and bursting pain in theleft submandibular region. The swellingappeared after eating spicy foods. Aftera physical examination, palpation and X-ray the patient was diagnosed with sialoli-thiais of submandibular gland with theconcrement localized in the anterior duct.What tactics should be chosen by a dentalsurgeon after removal of the concrementfrom the duct?

A. The wound and the duct are not to besutured and drainedB. The wound is to be sutured along theline of incisionC. The wound is to be sutured along theline of incision and drainedD. Plasty of the excretory ductE. The excretory duct and the wound areto be sutured

152. A 14-year-old child has been latelycomplaining about overgrowth andpermanent bleeding of gums during toothbrushing. Objectively: gingival papillae inthe anterior part of both jaws are swollen,hyperemic, overlap the tooth crowns by1/3 of their height, bleed easily during theinstrumental examination. Caries intensi-ty is of average level. Hygiene of the oralcavity is not satisfactory. In this case itis advisable to recommend a toothpastecontaining:

A. Aluminum compoundsB. Zinc compoundsC. Calcium compoundsD. Fluorine compoundsE. Pyrophosphate

153. During examination of a 5-year-old child the orthodontist revealed nowear of teeth, no tremata and diastemata,orthogenic occlusion. Which of thefollowing symptoms in a 5-year old chi-ld is a sign of future teeth overcrowding?

A. Absence of tremata and diastemataB. Absence of wear of teethC. Orthogenic occlusionD. Orthognathic biteE. Absence of mesial step in the region ofsecond temporary molars

154. An 8-year-old child has been cli-nically diagnosed with exacerbation ofchronic periodontitis of the 84 tooth. Thecrown is decayed by 1/2. What is the denti-st’s optimal tactics?

A. ExtractionB. Endodontic treatmentC. Endodontic treatment and drug therapyD. Drug therapyE. Opening along the mucogingival fold,drug thrapy

155. A 21-year-old patient complains ofblood that oozes from a cavity in an upperjaw tooth during eating. Objectively: theapproximal occlusal surface of the 27tooth has a deep carious cavity filled wi-th soft tissue overgrowth of purple-redcolour that bleeds easily when touchedby the probe. Probing causes a slight pain.There is a slight pain reaction to cold sti-muli. EPT result is 60 mA. Ro-gram showsno pathological changes. What is the mostlikely diagnosis?

A. Chronic hypertrophic pulpitisB. Hypertrophic papillitisC. Chronic granulating periodontitisD. Chronic fibrous periodontitisE. Chronic gangrenous pulpitis

156. A 20-year-old female patientcomplains of sore gums, purulent di-scharges from under the gums, toothmobility. Objectively: the gums aremarkedly hyperemic, cover the toothcrowns by 1/3. Periodontal pockets are7-8 mm deep with moderate purulent di-scharge. The 16, 26, 36, 31, 46, 41 teethare missing. The remaining teeth arecharacterized by the 2 grade mobility. Ro-gram shows root exposure by 2/3, the deep

Page 22: Krok 2 - 2013 Question Paper (Stomatology)

Krok 2 Stomatology 2013 22

bone niches in the alveolar bone. The skinkeratinization of the extensor surfaces ofthe elbows and knees is present, as well aslimb keratoderma. Nails are rough, dull.What disease can be suspected?

A. Papillon-Lefevre syndromeB. Generalized periodontitisC. Cyclic neutropeniaD. Eosinophilic granulomaE. Gomphosis

157. Examination of a 5-year-old childrevaled reverse overlap of the incisors andcanines. What is the most effective way ofabnormal bite prevention at this age?

A. Selective grinding of milk teethtuberclesB. Vestibular plateC. MyogymnasticsD. Tongue frenulum plastyE. Treatment with an orthodontic appli-ance

158. A 67-year-old patient consulted adoctor about a formation on his upper lipon the left, which appeared two monthsago. Objectively: on the border of skinand vermilion border of the upper lip onthe left there is a roundish cone-shapedformation up to 1 cm at the base, narrowi-ng towards the apex, up to 1.3 cm high,gray, dense, painless. What is the most li-kely diagnosis?

A. Cutaneous hornB. KeratoacanthomaC. FibromaD. NeuromaE. Nevus

159. A 13-year-old child has been takento an orthodontist. Objectively: the chi-ld has permanent dentition. The 13 toothwith mesio-distal dimensions of 8,9 mmerupted vestibularly outside the dentalarch, the interval between the 14 and 12tooth is 9,2 mm. Select a rational plan oftreatment:

A. Moving the 13 tooth into its properplace in the dental archB. Extraction of the14 tooth, moving the13 tooth in the distal direction into thedental archC. Expansion of the dental arch and movi-ng the 13 toothD. Finger massage of the 13 toothE. Extraction of the 13 tooth

160. A 7,5-year-old child has squaredental arches of both jaws, the relati-

onship between the canines and the fi-rst permanent molars corresponds withAngle’s class I. Specify the dental anomalyaccording to Kalvelis classification:

A. Anomaly of the dental arch shapeB. Anomaly of dental arch developmentC. Anomalies of individual teethD. MalocclusionE. Anomalies of jaw size

161. A 12-year-old child has half retenti-on of the 25 tooth, the dental arch lacksspace for for 1/3 of the crown. The upperincisors overlap the lower ones by 2/3, therelationship of the first permanent molarson the left corresponds with Angle’s class2. Rational design of orthodontic appli-ance for the upper jaw should include:

A. Unilateral screw for the 25 tooth andbite plateB. Bilateral expansion screwC. Radial symmetric screwD. Radial asymmetric screwE. Lateral inclined plane

162. A 2-month-old baby has acutepneumonia complicated by an abscess.In course of treatment the baby hasdeveloped soft tissue swelling of the leftsupraorbital and infraorbital region due toan edema and inflammatory infiltrate. Thechild is in grave condition. What compli-cation has developed?

A. Acute hematogenous osteomyelitis ofthe left upper jawB. Facial vein thrombophlebitisC. Acute dacryocystitisD. Nonodontogenic abscess of the leftinfraorbital regionE. Nonodontogenic phlegmon of the leftcheek

163. Objective examination of a 4-year-old child revealed caries in the 74, 84teeth, abnormal attachment of the upperlip frenulum, which does not require surgi-cal correction. The value of Fedorov-Volodkina oral hygiene index is 1,8. Howmany times a year the child should see adentist if the average caries intensity inthe respective age group in the given regi-on is 3?

A. OnceB. TwiceC. 2-3 timesD. 3 timesE. 3-4 times

164. A 5-year-old child complains of

Page 23: Krok 2 - 2013 Question Paper (Stomatology)

Krok 2 Stomatology 2013 23

spontaneous pain in an upper jaw toothon the right that is getting worse at ni-ght and during eating cold food. Objecti-vely: the 65 tooth has a deep carious cavi-ty communicating with the tooth cavity.Probing is painful, percussion is painless.Cold water causes long-lasting pain. Whatis your provisional diagnosis?

A. Exacerbation of chronic pulpitisB. Acute periodontitisC. Exacerbation of chronic periodontitisD. Acute serous pulpitisE. Acute purulent pulpitis

165. A 56-year-old patient has indicati-ons for a clasp denture for the upper jaw.Objectively: the 15, 16, 17, 18, 25, 26, 27, 28teeth are missing. What are the constructi-on peculiarities of the denture in thiscase?

A. Indirect retainersB. Reduced number of clasp elementsC. Increased occlusal surface of artificialteethD. Denture bar is located in the anteriorthird of palateE. Rigid clasp elements

166. A 30-year-old patient consulted adentist about dull pain, numbness in thelower teeth, the lower lip and chin onthe right. Anamnesis data: the problemsarose a week ago after the extraction of animpacted wisdom tooth on the right lowerjaw. The patient developed the followingdisease:

A. Neuritis of the inferior alveolar nerveB. Neuralgia of the third branch of thetrigeminal nerveC. Neuritis of the facial nerveD. Neuralgia of the auriculotemporalnerveE. Neuritis of the trigeminal nerve

167. A 56-year-old male patient complai-ns of tooth hypersensitivity to all kindsof stimuli. Objectively: gingival mucosa isanemic, thinned, dense on touch, toothnecks and roots are exposed by 1/3 oftheir length, ache when touched with atool. Teeth are stable. There are wedge-shaped defects in the cervical region of allthe premolars. What kind of toothpasteshould be advised this patient for thecomplex therapy of the disease?

A. Gel containing microelementsB. Paste containing salt additivesC. Paste containing herbal extractsD. Paste containing proteolytic enzymesE. Paste containing chlorhexidine

168. A 38-year-old female complains ofexperiencing pain in her lower lip forhalf a year, difficult eating and talki-ng. The patient sought medical help atplace of residence, but the administeredtreatment was ineffective. Examination ofthe lower lip revealed a deep fissure inthe centre which affected the vermilionborder and partly the mucous membraneof lips. The fissure was covered with abrown crust, after its removal the fissurestarted bleeding. The fissure edges arehyperkeratinized, infiltrated. Select theoptimal treatment tactics:

A. Fissure excisionB. Application of keratoplastic agentsC. Irradiation with helium-neon laserD. Application of corticosteroid ointmentE. Application of antibiotic ointment

169. An 18-year-old patient lives in anarea with fluoride concentration in thedrinking water at the rate of 0,7 mg/l.The patient complains of white spots onthe front teeth, sensitivity to sweet food.Objectively: there are chalky spots on thevestibular surface in the cervical part ofteeth. Enamel has lost its gloss. Which ofthe following methods will allow to makea diagnosis?

A. Vital stainingB. RadiographyC. Thermal testD. ProbingE. Electric pulp test

170. A 28-year-old patient has beenscheduled for the surgical resection ofthe root apex of the 12 tooth for chronicgranulomatous periodontitis. What kindof anesthesia should be given the patientbefore the surgery?

A. Bilateral infraorbital and incisiveB. Infraorbital and palatineC. Plexual and palatineD. Tuberal and palatineE. Terminal and incisive

171. During a dental visit a 50-year-old patient suddenly developed tinni-tus, nausea, dizziness, blackout, loss ofconsciousness, dramatic paleness, myosis.There were no pupillary reaction to lightand corneal reflexes. Heart sounds were

Page 24: Krok 2 - 2013 Question Paper (Stomatology)

Krok 2 Stomatology 2013 24

muffled, blood pressure was decreased.The described symptoms correspond withthe following medical emergency:

A. SyncopeB. Anaphylactic shockC. Hypertensive crisisD. CollapseE. Myocardial infarction

172. A 42-year-old female patientpresented to a dentist for oral sanitation.She has a history of tonsillitis, rheumatoidarthritis, Botkin’s disease. What are theconditions of instrument treatment afterthe patient’s visit?

A. Under the special schemeB. Under general conditionsC. Dry-air sterilizationD. Processing with lysolE. Processing with 0,1% solution ofchloramine

173. A 22-year-old patient complains ofdiscolouration of the 11 tooth. Objecti-vely: the 11 tooth is pink, hard tissuesare intact, percussion is painless, mucousmembrane around the tooth is unchanged.Radiography shows that the 11 tooth rootcanal is filed to the top, there are nochanges in the periapical tissues. The deepincisal overlap is present. What is the mostsuitable construction in this clinical si-tuation?

A. Porcelain veneerB. Combined Belkin crownC. Porcelain-fused-to-metal crownD. Metal-plastic crownE. Plastic crown

174. An 8,5-year-old girl complains of painin the lower right molar during eating.Objectively: the occlusal surface of the 85tooth exhibits a carious cavity filled wi-th softened dentin. The cavity is locatedwithin circumpulpal dentin and communi-cates with the tooth cavity. Probing causesacute pain, moderate bleeding. Percussionis painless. What is the optimal treatment?

A. Non-vital amputationB. Non-vital extirpationC. Vital extirpationD. Vital amputationE. Biological method

175. A 23-year-old patient complai-ns of minor root exposure, gingivalhaemorrhage during tooth brushing, gumitch. Objectively: there is supragingivaland subgingival dental calculus. Gums

are hyperaemic, edematic, pockets are 3,5mm deep. X-ray picture shows resorptionof interalveolar septa by 1/3. What is themost likely diagnosis?

A. Chronic generalized I degree peri-odontitisB. Chronic generalized II degree peri-odontitisC. Exacerbation of generalized I degreeperiodontitisD. Exacerbation of generalized II degreeperiodontitisE. II degree periodontitis

176. A 34-year-old male patient complainsof a cosmetic defect, a cavity on the vesti-bular surface in the cervical part of the21 tooth. Objectively: the carious cavity iswithin the enamel, the floor and the wallsare pigmented, probing and percussionand painless. There is no pain reaction tostimuli. What is the most likely diagnosis?

A. Chronic surface cariesB. Acute surface cariesC. Necrosis of dental hard tissuesD. Acute median cariesE. Chronic median caries

177. A 14-year-old patient complains ofintense, throbbing pain in the region of the46 tooth which appeared for the first timetwo days ago. Objectively: the 46 toothis filled. Percussion causes acute pain, thetooth is mobile, the mucous membranearound the 46 tooth is hyperemic andedematous, palpation of the mucogingivalfold in the root apex projection is painful.Ro-gram shows no changes in the peri-apical region of the 46 tooth. What is themost likely diagnosis?

A. Acute purulent periodontitisB. Acute localized periodontitisC. Acute purulent pulpitisD. Acute serous periodontitisE. Exacerbation of chronic periodontitis

178. Mother of a 1,5-year-old childcomplains of a formation on the child’s ri-ght cheek. This formation appeared a fewmonths after birth. Objectively: on the ri-ght cheek there is a crimson formationsized 1,5-2,0 cm, painless, with a positivesign of filling. What is the most likely di-agnosis?

Page 25: Krok 2 - 2013 Question Paper (Stomatology)

Krok 2 Stomatology 2013 25

A. Cavernous hemangioma of the rightcheekB. ErysipelasC. Hematoma of the right cheekD. Vascular nevusE. Lymphangioma of the right cheek

179. A 28-year-old patient complains ofpain and bleeding of gums in the frontalpart of the upper jaw on the left. Twoyears ago, the 22 tooth was covered witha porcelain-fusd-to-metal crown. Objecti-vely: interdental papilla between the 21and 22 tooth is hypertrophied, markedlyhyperemic, overlaps the crown of the 22by 1/3 of its height, bleeds when touched.Periodontal pocket between the 21 and22 tooth is 4 mm deep. Artificial crown islocated on gingival margin. Radiographyreveals resorption of the interalveolarsepta between the 21 and 22 tooth by 1/3of their height. Specify a priority action inthe treatment of this patient:

A. Removal of the artificial crownB. GingivectomyC. Anti-inflammatory therapyD. Curettage of the periodontal pocketE. Sclerotherapy

180. A 13-year-old girl complains of gingi-val haemorrhages during tooth brushingand eating solid food. Objectively: the gi-ngival mucosa of the upper and lower jawsis somewhat edematic, hyperemic. Theoral hygiene is inadequate. X-ray showsno changes. Make a diagnosis:

A. Chronic catarrhal gingivitisB. Acute catarrhal gingivitisC. Hypertrophic gingivitisD. Desquamative gingivitisE. Chronic atrophic gingivitis

181. A 45-year-old patient undergoesteeth preparation. 15 minutes afteranesthetization with 4% solution ofUbistesin forte the patient developedhyperemia of skin, increased heart rate,headache, syncope. Previously the pati-ent had not exhibited such reaction to thisanesthetic. What complication occurred inthe patient?

A. Arterial pressure riseB. Anaphylactic shockC. SyncopeD. CollapseE. Myocardial infarction

182. A 23-year-old patient complains ofpain and swelling in the region of a centralincisor which was injured some time ago.

After clinical and radiographic exami-nation the patient has been diagnosedwith acute purulent periostitis of theupper jawmaxilla extending from the 11tooth; radiograph shows a slight wideni-ng of periodontal ligament space of the11 tooth. Choose the best method oftreatment:

A. Subperiosteal abscess opening,endodontic treatment of the 11 toothB. Endodontic treatment of the 11 tooth,anti-inflammatory therapyC. Extraction of the 11 toothD. Extraction of the 11 tooth, anti-inflammatory therapyE. Extraction of the 11 tooth, subperiostealabscess opening

183. A 45-year-old patient presented to aprosthodontics clinic. During the objecti-ve examination the doctor checked thesagittal movements of the lower jaw.What muscles are responsible for sagittalmovements of the lower jaw?

A. Lateral pterygoid musclesB. Medial pterygoid musclesC. Mandibulohyoid muscleD. Digastric muscleE. Mentohyoid muscle

184. A 55-year-old patient presented to aprosthetic dentistry clinic for prostheti-cs. During the patient examinationthe movements of the lower jaw werechecked. Specify the position of the headsof mandible relative to the articulartubercle with maximal mouth opening:

A. At the top of the articular tubercleB. The heads slide over the surface of thearticular tubercleC. At the antrior edge of the articulartubercleD. At the base of the articular tubercleE. At the posterior edge of the articulartubercle

185. After examination a 42-year-oldpatient has been diagnosed with acutepurulent periostitis of the lower jawextending from the 35 tooth. What is themost rational method of anesthesia forthe 35 tooth extraction and lancing thesubperiosteal abscess?

A. Torusal (Weisbrem’s)B. BerchetC. InfiltrationD. PlexualE. General anesthesia

Page 26: Krok 2 - 2013 Question Paper (Stomatology)

Krok 2 Stomatology 2013 26

186. A 10-year-old girl complains ofbleeding from the alveolar socket of theextracted 63 tooth lasting for three days.The patient has a history of prolongedbleeding after removal of primary teethand frequent nosebleeds. Objectively:oral mucosa is pale, there are smallhemorrhages. The socket of the extracted63 tooth is filled with a big bulging clotwith blood oozing from under it. Forearmskin has petechiae. What specialist shouldbe visited in the first place?

A. HematologistB. NeuropathistC. EndocrinologistD. Infectious diseases specialistE. Gastroenterologist

187. A 20-year-old male patient withinsulin-dependent diabetes mellitus hasbeen given anesthesia with Ultracain DS-forte for surgical oral sanitation. Whatcomplications may develop in this case?

A. Abnormal levels of blood glucoseB. Increased blood pressureC. ClonusD. Atopic dermatitisE. Toxic shock

188. A 5-year-old boy has been di-agnosed with acute mumps. The chi-ld is in a moderately grave condition,body temperature is of 38, 6oC, the pati-ent complains of acute pain in the lefthypochondrium and epigastrium, radiati-ng to the back. What complication hasdeveloped in the child?

A. PancreatitisB. OrchitisC. EpididymitisD. MediastinitisE. Gastritis

189. A 6-year-old girl with acute purulentodontogenic lymphadenitis had been gi-ven intramuscular injection of cefazolin(0,5 twice a day). 1,5 hour after thedrug injection the child developed edemaof lips, cheeks, forehead, ears, mucousmembrane of mouth and upper airways.What complication developed in the girl?

A. Angioneurotic edemaB. UrticariaC. Allergic dermatitisD. Anaphylactic shockE. Toxic shock

190. As a result of an injury the 51and 61teeth of a 3-year-old child have completely

cut into the alveolar bone. History record:the teeth were partially decayed, changedin colour. Select an appropriate treatmenttactics:

A. Extraction of the injured teethB. Follow-up as the teeth may erupt againC. Reposition of teeth, endodontictreatmentD. Extraction of teeth, endodontictreatment, repositionE. Treatment is not required

191. A 43-year-old patient complains ofmobility and significant neck exposureof the lower front teeth. Objectively: thegums in the region of the 44, 43, 42, 41, 31,32, 33, 34 teeth are pale and cyanotic, non-bleeding. The 42, 41, 31, 32 teeth exhibitthe I-II grade mobility. The overcrowdi-ng of the 42, 41, 31, 32 teeth is present.The necks of the 42, 41, 31, 32 teeth areexposed by 1/2 of the root length, thenecks of the 43, 33 teeth are exposed by1/4. What kind of dental prosthesis shouldbe applied in this case?

A. Cast removable splintB. Kurlyandsky splint barC. Cap splintD. Portion crownE. Half-ring splint

192. A 7-year-old child has to undergoplastic surgery of the upper lip frenulum.What operation should be performed tolengthen the frenulum of the upper lip?

A. Dieffenbach graftingB. Thiersch grafting with local flapsC. Relaxing incisions of the mucosaD. Grafting with a pedicle flapE. Szymanowsky grafting with local flaps

193. Routine examination of a 9-year-old girl revealed symmetrical horizontalgrooves on the vestibular surfaces of the16, 12, 11, 21, 22, 26, 31, 32, 36, 41, 42, 46teeth. The grooves alternated with intacttooth tissues and were present in theseteeth at the time of their eruption. Thepatient has a history record of pneumoniaat the age of 10 months treated with acourse of antibiotics. Make a diagnosis:

A. Systemic hypoplasiaB. Local hypoplasiaC. Focal hypoplasiaD. Dental fluorosisE. Initial caries

194. A patient with a relevant historyof allergic reactions had undergone tests

Page 27: Krok 2 - 2013 Question Paper (Stomatology)

Krok 2 Stomatology 2013 27

for allergic sensitivity to metals. Accordi-ng to indications, the bridges of preciousmetal (gold) were fabricated. Specify thewarranty period starting from the day offinishing the prosthetic procedures?

A. 12 monthsB. 3 monthsC. 6 monthsD. 9 monthsE. 18 months

195. A 60-year-old patient needs a parti-al removable denture for the upper jaw.The central occlusion has been determi-ned. What is the next stage of prosthetics?

A. Checking the wax denture constructionB. Try-in of the dentureC. Fabrication of plaster modelsD. Relining the dentureE. Taking impressions

196. A 12-year-old child complains ofburning lips, pain at mouth opening,yellowish crusts on the vermilion border.Objectively: there are yellowish crusts onthe vermilion border extending from onemouth corner to another, from the wet-dryline to the middle of the vermilion border.When the crusts are removed, the smoothbright red moist surface can be seen. Whatis the most likely diagnosis?

A. Exfoliative cheilitisB. Atopic cheilitisC. Meteorological cheilitisD. Eczematous cheilitisE. Glandular cheilitis

197. Parents of a 3-year-old child took thechild to a pedodontist for oral sanitati-on. The child has cerebral palsy. Objecti-vely: there is multiple dental caries, gingi-val mucosa is hyperemic, oral hygiene is

inadequate. What method of dental healthcare may be appropriate in this case?

A. Under general anesthesiaB. Under local anesthesia injectionC. Under local application anesthesiaD. Without anesthesiaE. Giving sedatives

198. A patient with ischemic heart di-sease has developed ventricular fibrillati-on. What is the first-priority therapeuticaction?

A. Electric defibrillationB. Lidocaine injectionC. Adrenaline injectionD. Potassium chloride injectionE. Novocaine amide injection

199. After an injection of 30 IU of inslulina diabetic patient developed a short-termexcitation followed by loss of consci-ousness. What is the most likely cause?

A. Hypoglycemic comaB. StrokeC. Hyperglycemic comaD. Myocardial infarctionE. Ketoacidotic coma

200. During the blood transfusion a pati-ent developed seizures, tremor, lumbaland retrosternal pain. Objectively: skinbecame hyperemic and later turned pale,the patient had hyperhydrosis, labouredbreathing, Ps at the rate of 110/min, AP-70/40 mm Hg, black urine. What compli-cation occurred in the patient?

A. Transfusion shockB. Acute renal failureC. Pulmonary embolismD. Anaphylactic shockE. Hypotensive crisis